Please confirm topic selection

Are you sure you want to trigger topic in your Anconeus AI algorithm?

Please confirm action

You are done for today with this topic.

Would you like to start learning session with this topic items scheduled for future?

Review Question - QID 213801

In scope icon M 7 A
QID 213801 (Type "213801" in App Search)
A 58-year-old man presents to the emergency department with shortness of breath. He states his symptoms have been worsening since he was discharged from the hospital 3 days ago after being treated for a myocardial infarction. The patient has a past medical history of diabetes and chronic obstructive pulmonary disease and is an active smoker. His temperature is 98.4°F (36.9°C), blood pressure is 155/94 mmHg, pulse is 105/min, respirations are 16/min, and oxygen saturation is 90% on room air. Physical exam is notable for bilateral crackles and wheezes. An initial ECG demonstrates pathologic Q waves. A chest radiograph is ordered as seen in Figure A. An initial troponin is 0.9 ng/mL and 0.9 ng/mL on repeat 4 hours later. Which of the following is the most appropriate treatment for this patient?
  • A
  • A